Quick Question- Hamiltonian constant proof

Click For Summary
The discussion focuses on proving that the Hamiltonian is a constant of motion when the Lagrangian does not explicitly depend on time. A participant seeks clarification on the transition from the differential form of the Hamiltonian to its time derivative, questioning the absence of the product rule in their calculations. Another participant explains that the differentiation process was already accounted for in the previous step, emphasizing that transitioning from dH to dH/dt involves division by dt rather than differentiation. The conversation highlights the importance of understanding the mathematical distinctions between differentials and derivatives in this context. Overall, the thread provides insights into the nuances of Hamiltonian mechanics and Lagrangian dynamics.
binbagsss
Messages
1,291
Reaction score
12

Homework Statement



Show that if the Lagrangian does not explicitly depend on time that the Hamiltonian is a constant of motion.

Homework Equations



see below

The Attempt at a Solution


method attached here:
hcom.png


Apologies this is probably a bad question, but just on going from the line ##dH## to ##dH/dt## I see the##d/dt## has hit the ##dq_0## terms only, I don’t understand why a product rule hasn’t been used, so I would get:##\frac{dH}{dt}=\sum_{u} \dot{q_u} (\frac{dp_{u}}{dt}-\frac{\partial L}{\partial q_u} )+ \ddot{q_u}dp_{u}-\frac{d}{dt}(\frac{\partial L}{\partial q_u}) dq_u ##Many thanks
 

Attachments

  • hcom.png
    hcom.png
    17.4 KB · Views: 730
Physics news on Phys.org
You are not taking the derivative of ##dH##. ##dH## in itself is a differential
 
In going from dH to dH/dt, you are not differentiating, but dividing by dt. Differentiation was in the previous step.
Thus e.g. if f = uv
df = udv + vdu
df/dx = udv/dx + vdu/dx (not uddv/dx + dv du/dx +vddu/dx + du dv/dx)

Edit: Beat me to it!
 
  • Like
Likes binbagsss
Question: A clock's minute hand has length 4 and its hour hand has length 3. What is the distance between the tips at the moment when it is increasing most rapidly?(Putnam Exam Question) Answer: Making assumption that both the hands moves at constant angular velocities, the answer is ## \sqrt{7} .## But don't you think this assumption is somewhat doubtful and wrong?

Similar threads

Replies
4
Views
2K
Replies
6
Views
2K
  • · Replies 3 ·
Replies
3
Views
2K
  • · Replies 2 ·
Replies
2
Views
593
  • · Replies 3 ·
Replies
3
Views
1K
  • · Replies 4 ·
Replies
4
Views
2K
Replies
8
Views
2K
  • · Replies 3 ·
Replies
3
Views
886
  • · Replies 5 ·
Replies
5
Views
2K
  • · Replies 4 ·
Replies
4
Views
2K